How to Prove that l is the Least Upper Bound of a Set S of Real Numbers?

  • Thread starter Thread starter roam
  • Start date Start date
  • Tags Tags
    Continuity Limit
Click For Summary
To prove that l is the least upper bound (l.u.b.) of a nonempty set S of real numbers, one must show that for every ε > 0, there exists an x in S such that x > l - ε. The discussion emphasizes using proof by contradiction, suggesting that if all x in S were less than or equal to l - ε, then l - ε would also serve as an upper bound, contradicting the definition of l as the least upper bound. Participants express confusion about the problem's phrasing and the relevance of limits, clarifying that the focus should remain on the properties of l and S. Overall, the key takeaway is the necessity of demonstrating that no upper bound can be less than l while maintaining that l is indeed the least upper bound.
roam
Messages
1,265
Reaction score
12
Let l \in R be the least upper bound of a nonempty set S of real numbers.
Show that for every \epsilon < 0 there is an x \in S such that
x > l - \epsilon




I don't understand this question very well, I appreciate it if you could give me some hints.



l is the l.u.b on S, therefore it is greater than or equal to any s \in S

By the definition of the limit; |f(x)-l| < ε if 0 < |x-a| < δ

l-ε < x < l+ε

|f(x)-l| < ε

"?"
 
Last edited:
Physics news on Phys.org
Suppose it's NOT true. Then there is an epsilon such that x<=l-epsilon for all x in S. Can l really be lub(S)?
 
Thanks, so that's how you derive the contradiction.

But could you explain it in more detail please(especially the last bit)?

I appreciate that.
 
l is supposed to be the LEAST upper bound. If x<=l-epsilon for all x in S, then l-epsilon is ALSO an upper bound. What the order relation between l and l-epsilon?
 
roam said:
Let l \in R be the least upper bound of a nonempty set S of real numbers.
Show that for every \epsilon &lt; 0 there is an x \in S such that
x &gt; l - \epsilon




I don't understand this question very well, I appreciate it if you could give me some hints.



l is the l.u.b on S, therefore it is greater than or equal to any s \in S

By the definition of the limit; |f(x)-l| < ε if 0 < |x-a| < δ

What limit and what f(x) are you talking about? There is no function nor limit of a function mentioned in the problem

l-ε < x < l+ε

|f(x)-l| < ε

"?"
Dick has given the hint you need: use proof by contradiction.
 
Question: A clock's minute hand has length 4 and its hour hand has length 3. What is the distance between the tips at the moment when it is increasing most rapidly?(Putnam Exam Question) Answer: Making assumption that both the hands moves at constant angular velocities, the answer is ## \sqrt{7} .## But don't you think this assumption is somewhat doubtful and wrong?

Similar threads

  • · Replies 1 ·
Replies
1
Views
2K
Replies
7
Views
1K
  • · Replies 3 ·
Replies
3
Views
2K
  • · Replies 2 ·
Replies
2
Views
2K
  • · Replies 1 ·
Replies
1
Views
1K
  • · Replies 13 ·
Replies
13
Views
4K
  • · Replies 8 ·
Replies
8
Views
2K
Replies
7
Views
2K
  • · Replies 33 ·
2
Replies
33
Views
2K
  • · Replies 25 ·
Replies
25
Views
3K